Pergunta

Let $\to_\beta$ be $\beta$-reduction in the $\lambda$-calculus. Define $\beta$-expansion $\leftarrow_\beta$ by $t'\leftarrow_\beta t \iff t\to_\beta t'$.

Is $\leftarrow_\beta$ confluent? In other words, do we have that for any $l,d,r$, if $l \to_\beta^* d\leftarrow_\beta^* r$, then there exists $u$ such that $l\leftarrow_\beta^* u \to_\beta^* r$?

Keywords: Upward confluence, upside down CR property


I started by looking at the weaker property: local confluence (i.e. if $l \to_\beta d\leftarrow_\beta r$, then $l\leftarrow_\beta^* u \to_\beta^* r$). Even if this were true, it would not imply confluence since $\beta$-expansion is non-terminating, but I thought that it would help me understand the obstacles.

(Top) In the case where both reductions are at top-level, the hypothesis becomes $(\lambda x_1.b_1)a_1\rightarrow b_1[a_1/x_1]=b_2[a_2/x_2]\leftarrow (\lambda x_2.b_2)a_2$. Up to $\alpha$-renaming, we can assume that $x_1\not =x_2$, and that neither $x_1$ nor $x_2$ is free in those terms.

(Throw) If $x_1$ is not free in $b_1$, we have $b_1=b_2[a_2/x_2]$ and therefore have $(\lambda x_1.b_1)a_1=(\lambda x_1.b_2[a_2/x_2])a_1\leftarrow(\lambda x_1.(\lambda x_2.b_2)a_2)a_1\rightarrow (\lambda x_2.b_2)a_2$.

A naive proof by induction (on $b_1$ and $b_2$) for the case (Top) would be as follows:

  • If $b_1$ is a variable $y_1$,

    • If $y_1=x_1$, the hypothesis becomes $(\lambda x_1.x_1)a_1\rightarrow a_1=b_2[a_2/x_2]\leftarrow (\lambda x_2.b_2)a_2$, and we indeed have $(\lambda x_1.x_1)a_1=(\lambda x_1.x_1)(b_2[a_2/x_2])\leftarrow (\lambda x_1.x_1)((\lambda x_2.b_2)a_2)\rightarrow (\lambda x_2.b_2)a_2$.

    • If $y_1\not=x_1$, then we can simply use (Throw).

  • The same proofs apply is $b_2$ is a variable.

  • For $b_1=\lambda y.c_1$ and $b_2=\lambda y.c_2$, the hypothesis becomes $(\lambda x_1.\lambda y. c_1)a_1\rightarrow \lambda y.c_1[a_1/x_1]=\lambda y.c_2[a_2/x_2]\leftarrow (\lambda x_2.\lambda y.c_2)a_2$ and the induction hypothesis gives $d$ such that $(\lambda x_1.c_1)a_1\leftarrow d\rightarrow (\lambda x_2.c_2)a_2$ which implies that $\lambda y.(\lambda x_1.c_1)a_1\leftarrow \lambda y.d\rightarrow \lambda y.(\lambda x_2.c_2)a_2$. Unfortunately, we do not have $\lambda y.(\lambda x_2.c_2)a_2\rightarrow (\lambda x_2.\lambda y.c_2)a_2$. (This makes me think of $\sigma$-reduction.)

  • A similar problem arises for applications: the $\lambda$s are not where they should be.

Nenhuma solução correta

Licenciado em: CC-BY-SA com atribuição
Não afiliado a cs.stackexchange
scroll top